(x^2+12x+36)-(x-1)^2
x^2-4xy+4y^2
16x^2-8x+1
x^2+4x+4
4x^2+12xy+9y^2
x^2-8x-16
4x^2-16
x^2-1
x^2+6x+9
9x^2-25y^4
16x^2-8x+1

Answers

Answer 1

Answer:

Hey I'm sorry I didn't get to answer your question it's just that I need the points because I don't have enough to get help with my question. I hope you get the answer that you need for you question. Good Luck :)

Step-by-step explanation:


Related Questions

Can some help me with 12 and 13 and 14

Answers

12. The answer is C.

Steel rods are manufactured with a mean length of 29 centimeter (cm). Because of variability in the manufacturing process, the lengths of the rods are approximately normally distributed with a standard deviation of 0.07 cm. (a) What proportion of rods has a length less than 28.9 cm? (b) b) Any rods that are shorter than 24.84 cm or longer than 25.16 cm are discarded. What proportion of rods will be discarded?

Answers

Solution :

Given data :

The mean length of the steel rod = 29 centimeter (cm)

The standard deviation of a normally distributed lengths of rods = 0.07 centimeter (cm)

a). We are required to find the proportion of rod that have a length of less than 28.9 centimeter (cm).

Therefore, P(x < 28.9) = P(z < (28.9-29) / 0.07)

                                    = P(z < -1.42)

                                   = 0.0778

b). Any rods which is shorter than [tex]24.84[/tex] cm or longer than [tex]25.16[/tex] cm that re discarded.

Therefore,

P (x < 24.84 or 25.16 < x) = P( -59.42 < z or -54.85)

                                         = 1.052

Which of the following is true of the discriminant for the graph below?

Answers

Considering that the quadratic equation has no solutions, the discriminant is classified as:

C. Negative.

What is the discriminant of a quadratic equation and how does it influence the solutions?

A quadratic equation is modeled by:

[tex]y = ax^2 + bx + c[/tex]

The discriminant is:

[tex]\Delta = b^2 - 4ac[/tex]

The solutions are as follows:

If [tex]\mathbf{\Delta > 0}[/tex], it has 2 real solutions.If [tex]\mathbf{\Delta = 0}[/tex], it has 1 real solutions.If [tex]\mathbf{\Delta < 0}[/tex], it has 2 complex solutions.

Looking at the graph, the equation has no solutions, hence [tex]\Delta < 0[/tex] and option C is correct.

More can be learned about the discriminant of a quadratic equation at https://brainly.com/question/19776811

#SPJ1

9/37 is changed to a decimal. What digit lies in the 2005th place to the right of the decimal point?

Answers

Answer:

2

Step-by-step explanation:

Divide 9/37 and you get repeating decimal of 0.243

Divide 2005 by 3 because the decimal repeats 3 numbers

You will get reminder of 1 from dividing 2005 by 3

Move 1 place from the decimal point and you get 2

Thank you so much for your help

Answers

Answer:

1.1x

Step-by-step explanation:

that is the procedure above

Let f(x) = 2x - 7 and g(x) = -6x - 3. Find f(x) + g(x) and state its domain.
HELP PLSSSSS!!!!!!!!!!!!!!!!!!!!!!!!!!!


A : 12x2 - 48x + 21; all real numbers
B: -14x2 + 36x - 18; all real numbers except x = 7
C: 12x2 - 48x + 21; all real numbers except x = 1
D: -14x2 + 36x - 18; all real numbers

Answers

Answer:

Step-by-step explanation:

f(x) + g(x) = 2x -  7 - 6x - 3

f(x) + g(x) = -4x - 10

The domain is any real number.

I think you have mixed up the question. None of the choices are correct. They look like they belong to another choice.

It could be f(x)*g(x)

(2x - 7) (-6x - 3)

-12x^2 - 42x - 6x + 32

-12x^2 - 48x + 21

Well it could be either A or C since they are identical.

Introduction to area of a piecewise rectangular figure

Answers

Given:

The piecewise rectangular figure.

To find:

The area of the piecewise rectangular figure.

Solution:

Draw a line and divide the given figure in two parts (a) and (b) as shown in the below figure.

Figure (a) is a rectangle of length 5 yd and width 3 yd. So, the area of the rectangle is:

[tex]Area=length\times width[/tex]

[tex]A_a=5\times 3[/tex]

[tex]A_a=15[/tex]

Figure (b) is a square of edge 2 yd. So, the area of the square is:

[tex]Area=(edge)^2[/tex]

[tex]A_b=(2)^2[/tex]

[tex]A_b=4[/tex]

The area of the given figure is:

[tex]A=A_a+A_b[/tex]

[tex]A=15+4[/tex]

[tex]A=19[/tex]

Therefore, the area of the given figure is 19 square yd.

Please need help explanation need it

Answers

Answer:

308 m^3

Step-by-step explanation:

The volume is given by

V = l*w*h  where l is the length , w is the width and h is the height

V = 7*4*11

V = 308 m^3

- CA Geometry A Illuminate Credit 4 FF.pdf

Answers

Answer:

hii

Step-by-step explanation:

If a 750 ml bottle of juice costs £1.90 and a 1 litre bottle of the same juice costs £2.50 then the 750 ml bottle is better value.

Answers

Answer:

The 1 liter bottle is better value

Step-by-step explanation:

Cost of 750 ml = £1.90

Cost of 1 liter = £2.50

1000 ml = 1 liter

Cost per 250 ml

750 ml / 3 = £1.90 / 3

250 ml = £0.6333333333333

Approximately,

£ 0.633

Cost per 250 ml

1 liter / 4 = £2.50 / 4

250 ml = £0.625

The 750 ml bottle is not a better value

The 1 liter bottle is better value

What is the period 3 pi and 4 pi

Answers

Answer:

i think i know the answer sorry if im wrong but i would say B

Step-by-step explanation:

Which of the following is a correct tangent ratio for the figure? A) tan (24) 76 B) tan(76°) °= 2 C) tan(76°) = D) tan(8") = 24 76​

Answers

Given question is incorrect; here is the complete question.

"Which of the following is a correct tangent ratio for the figure attached"

A) tan(76°) = [tex]\frac{24}{8}[/tex]

B) tan (76°) = [tex]\frac{8}{24}[/tex]

C) tan (24°) = [tex]\frac{76}{8}[/tex]

D) tan (8°) = [tex]\frac{24}{76}[/tex]

Option A will be the correct option.

   From the figure attached,

Given triangle is a right triangle.Measure of one angle = 76°Measure of two sides of the triangle are 24 and 8units.

By applying tangent ratio of angle having measure 76°.

tan(76°) = [tex]\frac{\text{Opposite side}}{\text{Adjacent side}}[/tex]

             = [tex]\frac{24}{8}[/tex]

    Therefore, Option (A) is the correct option.

Learn more,

https://brainly.com/question/14169279

Please help!

A line intersects the points (-2, 8) and
(4, 12). Find the slope and simplify
completely.
Help Resource
Slope
[?]
= +
Hint: m =
y2-yi
X2-X1
Enter

Answers

Answer:

2/3

Step-by-step explanation:

We can use the slope formula

m = (y2-y1)/(x2-x1)

 = (12-8)/(4 - -2)

  (12-8)/(4+2)

  4/6

   2/3

Determine the period

Answers

Answer:

3 units

Step-by-step explanation:

The period of a wave is the time taken to complete a cycle of motion of the wave

In the given figure, the graduations of the x-axis, which is the usually time axis = 1 unit

At the origin, (0, 0), the vertical displacement of the wave = 0

The maximum value of the wave function is between x = 0 and x = 1

The minimum value of the wave function is between x = 2 and x = 3

At the point (3, 0) the value of the wave function is again 0, and a cycle of the wave is completed

Therefore, the period of the wave = 3 units of the x-variable

convert 4 seconds to hour​

Answers

Answer:

0.00111111 hrs

Step-by-step explanation:

Have a nice day

Answer:

4/3600 = .001111 hr

Step-by-step explanation:

4 seconds * 1 hour            *    1 minute            =    4/3600 = .001111 hr

                    60 minutes          60 seconds

Solve. -7x+1-10x^2=0

Answers

Answer:

[tex]-7x+1-10x^2=0[/tex]

[tex]-10x^2-7x+1=0[/tex]

[tex]quadratic\:equation:-[/tex] [tex]ax^2+bx+c=0[/tex]

[tex]solutions:-\\\\x_{1,\:2}=\frac{-b\pm \sqrt{b^2-4ac}}{2a}[/tex]

[tex]For \\A=-10\\B=-7\\C=1[/tex]

[tex]x_{1,\:2}=\frac{-\left(-7\right)\pm \sqrt{\left(-7\right)^2-4\left(-10\right)\cdot \:1}}{2\left(-10\right)}[/tex]

[tex]\sqrt{\left(-7\right)^2-4\left(-10\right)\cdot \:1}=\sqrt{89}[/tex]

[tex]x_{1,\:2}=\frac{-\left(-7\right)\pm \sqrt{89}}{2\left(-10\right)}[/tex]

[tex]x_1=\frac{-\left(-7\right)+\sqrt{89}}{2\left(-10\right)},\:x_2=\frac{-\left(-7\right)-\sqrt{89}}{2\left(-10\right)}[/tex]

[tex]\frac{-\left(-7\right)+\sqrt{89}}{2\left(-10\right)}=-\frac{7+\sqrt{89}}{20}[/tex]

[tex]\frac{-\left(-7\right)-\sqrt{89}}{2\left(-10\right)}=\frac{\sqrt{89}-7}{20}[/tex]

[tex]x=\frac{\sqrt{89}-7}{20}[/tex]

OAmalOHopeO

On Monday morning at 8:00 a.m. the temperature is – 14 o C. Over the
next 6 hours the temperature rises 6 o C. Between 2:00 p.m. on Monday
and 8:00 a.m. on Tuesday the temperature drops 9 o C. Over the next 6
hours the temperature rises only 4 o C. What is the temperature at 2:00
p.m. on Tuesday?

Answers

Answer : the temperature is -13 degrees Celsius

PLS HELP QUESTION ATTACHED

Answers

Answer:

A

Step-by-step explanation:

the -1 represents the graph going down by 1

juans pencil box measures 6 cm long. if the length of the diagonal is 10 cm what is the width of the pencil box

Answers

Answer:

8 cm

Step-by-step explanation:

We can use the Pythagorean theorem to solve since we have a right triangle

a^2 +b^2 = c^2 where a and b are the legs and c is the hypotenuse

a^2 +6^2 = 10^2

a^2 +36 = 100

a^2 = 100-36

a^2 = 64

Taking the square root of each side

sqrt(a^2) = sqrt(64)

a =8

Answer:

8 cm

Step-by-step explanation:

Use the Pythagorean theorem- [tex]a^{2} +b^{2} =c^{2}[/tex]

leg a: 6cm

leg b: unknown

hypotenuse: 10cm

Therefore [tex]6^{2} +x^{2} =10^{2} = 36+x^{2} =100[/tex]

Subtract 36 to 100 to isolate the [tex]x^{2}[/tex].    [tex]x^{2} =64[/tex]

Square root both sides and get your answer of 8cm

Show Workings.
Question is in attached image.​

Answers

Answer:

A.]A chord of a circle of diameter 40 cm subtends an angle of 70° at the centre of the circle.

Solution given;

diameter [d]=40cm

centre angle [C]=70°

(a) Find the perpendicular distance be tween the chord and the centre of the circle.

Answer:

we have

the perpendicular distance be tween the chord and the centre of the circle=[P]let

we have

P=d Sin (C/2)

=40*sin (70/2)

=22.9cm

the perpendicular distance be tween the chord and the centre of the circle is 22.9cm.

(b) Using = 3.142, find the length of the minor arc.

Solution given;

minor arc=[tex]\frac{70}{360}*πd=\frac{7}{36}*3.142*40[/tex]

=24.44cm

the length of the minor arc. is 24.44cm.

B.]In the diagram, XZ is a diameter of the cir cle XYZW, with centre O and radius 15/2 cm.

If XY = 12 cm, find the area of triangle XYZ.

Solution given:

XY=12cm

XO=15/2cm

XZ=2*15/2=15cm

Now

In right angled triangle XOY [inscribed angle on a diameter is 90°]

By using Pythagoras law

h²=p²+b²

XZ²=XY²+YZ²

15²=12²+YZ²

YZ²=15²-12²

YZ=[tex]\sqrt{81}=9cm[/tex]

:.

base=9cm

perpendicular=12cm

Now

Area of triangle XYZ=½*perpendicular*base

=½*12*9=54cm²

the area of triangle XYZ is 54cm².

Answer:

Question 1

a)

d = 40 cm ⇒ r = 20 cm

Let the perpendicular distance is x.

Connecting the center with  the chord we obtain a right triangle with hypotenuse of r and leg x with adjacent angle of 70/2 = 35°.

From the given we get:

x/20 = cos 35°x = 20 cos 35°x = 16.383 cm (rounded)

b)

The minor arc is 70° and r = 20

The length of the arc is:

s = 2πr*70/360° = 2*3.142*20*7/36 = 24.437 cm (rounded)Question 2

Since XZ is diameter, the opposite angle is the right angle, so the triangle XYZ is a right triangle.

r = 15/2 cm ⇒ XZ = d = 2r = 2*15/2 = 15 cm

Find the missing side, using Pythagorean:

[tex]YZ = \sqrt{XZ^2 - XY^2} = \sqrt{15^2-12^2} = \sqrt{81} = 9[/tex]

The area of the triangle:

A = 1/2*XY*YZ = 1/2*12*9 = 54 cm²

Calculate the answer to the correct number of significant figures: (1.705 + 0.5067) / (0.2 * 1.243) = ______.


8.897


8.8966


8.9


9


8.90

Answers

Answer:

8.9

Step-by-step explanation:

they said to the sig. figure so since it's 8.8966, so the answer will be 8.9

The answer to the correct number of significant figures is 8.897, the correct option is A.

What are Significant Figures?

Significant figures is a positional notation, these are the digits that are required to understand the quantity of something.

The expression is

⇒(1.705 + 0.5067) / (0.2 * 1.243)

=2.2117/0.2486

=8.89662

≈ 8.897

To know more about Significant figures

https://brainly.com/question/14359464

#SPJ2

This drawing would be a step in finding which point of concurrency in a triangle?

Answers

Answer:

B. Orthocenter

Step-by-step explanation:

The orthocenter of a triangle is the point of intersection of the three altitudes (the perpendicular from a vertex to the side facing the vertex) of a triangle

The steps used in constructing the orthocenter includes the steps for drawing of the perpendicular to the sides of the triangle from a point which is the vertex opposite the side of the triangle

Therefore, in the given diagram, a perpendicular to the side KS is drawn from the (vertex) point W, by drawing an arc with center at W that intersects the side KS at two points. Rom the points of intersection of the arc drawn from W intersects KS, with the compass, two arcs are drawn to intersect at a point from which a line drawn to W, is perpendicular to KS

Therefore;

The correct option is B. orthocenter

True or False?
k = 3 over 4 is a solution to the inequality 12k + 2 < 12.
True
False

Answers

Answer:

False.

Step-by-step explanation:

...................

We know that k is 3/4. We can plug in 3/4 into k. 12(3/4) + 2 < 12. This gives us 36/48. 36/48 simplified is 3/4. Our equation is currently 3/4 + 2 < 12. We need common denominators to add 3/4 and 2. 2 with a denominator of 4 is is 8/4. 3/4 + 8/4 is 11/4. 11/4 < 12. 11/4 as a mixed number is 2 3/4. 2 3/4 as a decimal is 2.75. 2.75 < 12. 2.75 is less than 12. This is true.

Evaluate the expression 3(5 + 2)(7 - 2) using order of operations.

Answers

The answer is 36. Do the parentheses first and then multiply by 3

Answer:

105

Step-by-step explanation:

The order of operations is written as PEMDAS. These letters stand for:

-Parentheses

-Exponents

-Multiplication

-Division

-Addition

-Subtraction

We follow these steps in order to solve expressions efficiently. Now, we are going to use PEMDAS to evaluate the expression 3(5+2)(7-2) step by step.

3(7)(5)  The first step is to simplify the numbers in the parentheses.

There are no exponents, so we skip to the next step, multiplication.

(3*7)(5)

21(5)

105

PEMDAS is no longer needed because 105 has come out to be our answer.

I hope this helps you out! Have an an awesome day :3

Help PLEASE eeeeeeeeeeeeeeeee

Answers

Answer:

0.6 is the answer please

which statement must be true about line TU?

Answers

line tu is parallel to rs

Answer:

line TU has no slope in the diagram above

Can I know the answer for the above questions

Answers

Answer:

Step-by-step explanation:

what is 8/9 divide 2/3?

Answers

Answer:

4/3

Step-by-step explanation:

8/9 ÷ 2/3

Simplify the complex fraction.

4/3

Step-by-step explanation:

8/9 ÷ 2/3

Simplify

4/3 is the correct answer

Which statement is true regarding the angles in the figure below?

Triangle E D F. A diagonal line is drawn from point D to form an angle.
Angle D is an exterior angle because it shares a side with the triangle.
Angle D is an exterior angle because it is not inside the triangle.
Angle D is an exterior angle because it is formed by one side of the triangle and by extending another side of the triangle.
Angle D is not an exterior angle.

Answers

An exterior angle is an angle supplementary to one of the interior angles.  

In other words, an exterior angle is the angle between one of the sides, and the extension of an adjacent side.

In the given diagram, the angle D is measured from one of the sides, but not to the extension of an adjacent side.

Therefore angle D is not an exterior angle.

Option D is the correct one!

An exterior angle is an angle supplementary to one of the interior angles.  

We have given that,

Triangle E D F. A diagonal line is drawn from point D to form an angle.

Angle D is an exterior angle because it shares a side with the triangle.

Angle D is an exterior angle because it is not inside the triangle.

Angle D is an exterior angle because it is formed by one side of the triangle and by extending another side of the triangle.

Angle D is not an exterior angle.

What is the exterior angle?

an exterior angle is an angle between one of the sides and the extension of an adjacent side.

In the given diagram, the angle D is measured from one of the sides, but not to the extension of an adjacent side.

Therefore angle D is not an exterior angle.

To learn more about the exterior angle visit:

https://brainly.com/question/24242466

#SPJ5

Please help me with this on the image

Answers

Answer:

hope this may help u

q. no (a)= 60%

(b)=55%

Answer:

a) [tex]\frac{6}{10}[/tex] = [tex]\frac{3}{5}[/tex]  = .6 or 60%

b) [tex]\frac{11}{20}[/tex]  = .55 or 55%

Step-by-step explanation:

Count the boxes.

[tex]\frac{shaded}{total}[/tex]

Divide and multiply by 100

Other Questions
Date of NoteFace AmountInterest RateTerm of Notea.January 5 *$90,000 60 days b.February 15 *21,000 4 30 days c.May 1968,000 Use the pass compos with dj and a direct object pronoun and the cues provided in parentheses. Follow the model.Model: Tu vas lire ce livre? (la semaine dernire)Answer Je lai dj lu la semaine dernire.1. Tu vas aider ta mre dans le jardin? (ce matin)2. Tu vas nous attendre un peu? (toute la matine!)3. Tu vas repasser les vtements aujourdhui? (hier)4. Tu vas acheter les baguettes? (aprs lcole)5. Tes copains et toi, vous allez faire les soldes pour trouver des shorts? (samedi)6. Lylou et toi, vous allez prendre votre goter maintenant? ( 4h00)7. Tu vas acheter les billets de train pour aller chez tes grands-parents? (ce matin)8. Tu vas emmener ta sur la piscine? (avant-hier)9. Tu vas bientt apporter les glaces? (il y a 5 minutes) What are our duties towards the nation ? List them What are the characteristics of an estuary? Im new and i need help!!Please help me of you know the answers. Which Connects the ovary and uterus find the range of the function y = -x^(2) + 1y -1y -1y 1y 1 Choose ALL of the following functions that represent exponential decay. f(x) = 5(2/3)^x The enzyme lactate dehydrogenase is responsible for fermentation of pyruvate to form lactate. This process occurs in red blood cells and in muscle cells during intense exercise. Why is fermentation a necessary process for these cells to meet their energy needs What is the image point of (3,1)(3,1) after a translation left 3 units and down 5 units? A line of best fit must pass through all data points of a graph.True or False? AC. DF, and GI are parallel. Use the figure to complete the proportion.AD/DG = ?/EH (-32ux+19u^2x^3-12u^6x^7) divided by (-4u^2x^3) Two buses leave towns 576 kilometers apart at the same time and travel toward each other. One bus travels 12hslower than the other. If they meet in 3 hours, what is the rate of each bus?kmRate of the slower bus:Rate of the faster bus: what is vertebrate?? Help me with this, i'm confused. On a coordinate plane, rhombus W X Y Z has points (negative 3, 1), (1, 4), (5, 1), and (1, negative 2). Rhombus WXYZ is graphed on a coordinate plane. What is the area of the rhombus? Kristi finds a shirt for $27.99 at the store.The sign says that it is 25% off theoriginal price. Kristi must also pay the 8.5%sales tax. What is the cost of the shirtafter the sales tax? 1. Tim Berners-Lee was famous for his research in physics before he invented the World Wide Web. The number of days (d) is equalto the quotient of the number ofhours (h) and 24.A. d = h - 24B. h = 24 dC. h = 24 hD. h = 24 + h